Ist die Wellenfunktion von Partikeln in einem Gas gespreizt oder lokalisiert?

Für ein einzelnes freies Teilchen, das lokalisiert beginnt, breitet sich das Wellenfunktionspaket mit der Zeit aus, sodass das Teilchen weniger lokalisiert wird. Nehmen wir nun an, dass wir ein Gas dieser Teilchen in einer Box haben und sie kollidieren lassen (unter Verwendung eines gewissen Potentials): Wird sich die Wellenfunktion jedes Teilchens immer noch unbegrenzt ausbreiten oder wirken Kollisionen als Quelle der Dekohärenz und der Wellenfunktion? relokalisiert wieder? Ich habe beide Argumente von verschiedenen Kollegen gehört, sogar in Lehrbüchern. Hat jemand eine Computersimulation gemacht, die zeigt, was das beste Bild wäre?

Ich habe hier eine ähnliche Frage gestellt , aber keine schlüssigen Antworten erhalten, also denke ich, dass es ziemlich subtil sein muss. Ich hoffe, Sie bekommen hier eine gute kanonische Antwort.
Es wäre interessant, wenn Sie einige der verschiedenen Argumente in Lehrbüchern zitieren würden, die Sie gesehen haben.
Eine gute Antwort auf diese Frage würde die identische Natur von Partikeln anerkennen, eine zweite Quantisierung verwenden und die Zwei-Punkt-Dichtekorrelation für ein (ideales) Gas berechnen.
Dies ist ein höchst nicht triviales Problem. Ich würde vorschlagen, dass Sie damit beginnen, das Streuproblem für identische Teilchen zu lösen. Eine Diskussion des Problems fand ich in Landau, Quantum Mechanics , par. 137.
Verwandte: en.wikipedia.org/wiki/Anderson_localization , ein System mit Lokalisierung. Die Frage ist, ob es im Kontext zutrifft, an diesem Punkt muss ich sagen, dass die Frage zu weit gefasst ist. Es hat jedoch sicherlich immer noch meine +1 und könnte z. B. durch die Verwendung des Kommentars von @DanielSank eingeschränkt werden.

Antworten (7)

Einleitung: Wie definieren wir „lokalisiert“?

Für ein einzelnes Teilchen oder für mehrere nicht verschränkte Teilchen lässt sich anhand der Ausdrücke für die Wellenfunktionen leicht erkennen, ob sie lokalisiert oder delokalisiert sind. Zum Beispiel könnten Sie sagen, dass die Wellenfunktion exponentiell oder schneller für große abfällt x , also mit einem Formular wie ψ ( x ) e x / ξ mit einigen charakteristischen Längenskalen ξ , dann ist es lokalisiert, während so etwas wie eine ebene Welle (die als in der betrachtet werden kann ξ Grenze) ist delokalisiert.

Bei wechselwirkenden Teilchen entwickelt sich der Vielteilchen-Quantenzustand im Allgemeinen zu etwas, das zwischen den Teilchen verschränkt ist. Dann gibt es für ein einzelnes Teilchen keine Wellenfunktion mehr, und die Frage der Lokalisierung ist nicht mehr so ​​einfach. Beispielsweise ist der Zwei-Teilchen-Zustand ψ ( x 1 , x 2 ) e ( x 1 x 2 ) 2 lokalisiert oder delokalisiert?

Ein üblicher Weg, um diese Idee von lokalisierten/erweiterten Vielteilchensystemen zu verallgemeinern, besteht darin, das Konzept der Verschränkungsentropie (1) zu verwenden und zu fragen, ob eine bestimmte Region mit einer anderen entfernten Region des Systems verschränkt ist. Für ein eindimensionales System ist die Verschränkungsentropie:

S ( ρ EIN ) = Tr [ ρ EIN Protokoll ρ EIN ] , mit ρ EIN die reduzierte Dichtematrix für dieses System:

ρ EIN ( x 1 , x 2 , x N , x 1 ' , x 2 ' , x N ' ) = | x ich | > x 0 | x ich ' | > x 0   d x 1 d x N   d x 1 ' d x N '   ψ ( x 1 , , x N ) ψ ( x 1 ' , , x N ' )

Hier schauen wir uns eine Region an x 0 zu x 0 . Wenn S exponentiell klein ist, dann wird das System lokalisiert, und wenn nicht, wird es erweitert. Beachten Sie, dass wir von Partikeln zu Regionen übergegangen sind. Dies ist natürlicher, wenn man über Lokalisierung nachdenkt, aber für eine einheitliche Teilchendichte zu einem bestimmten Zeitpunkt impliziert die Lokalisierung des einen das andere.

Das Gefühl von „lokalisiert“, das wir jetzt haben, ist, dass für ein lokalisiertes System eine Messung an einem Punkt den Quantenzustand an einem weit entfernten Punkt nicht stört. Wenn Sie unter Verwendung dieses Standards die obigen Berechnungen an einem Zustand wie dem obigen Zwei-Teilchen-Zustand oder einem Ein-Teilchen-Ebenenwellenzustand durchführen, werden Sie feststellen, dass sie eine Verschränkungsentropie ungleich Null haben und erweitert sind. Allerdings ist ein Zustand wie ψ ( x 1 , x 2 ) e ( x 1 / ξ ) 2 e ( ( x 2 2 x 0 ) / ξ ) 2 lokalisiert werden würde, solange x 0 ξ .

Thermalisierung des Eigenzustands

Okay, mit diesen Ideen kann ich jetzt einfach die Antwort geben: Für ein Quantensystem von Teilchen in einer Box, die mit einer Hartschalenabstoßung interagieren, in einem hoch angeregten Zustand und mit verdünnter Grenze und im Gleichgewicht, ist die Verschränkungsentropie proportional zum Volumen des Systems.

Das bedeutet grob gesagt, dass jeder Punkt in der Box mit jedem anderen Punkt gleichermaßen verschränkt ist. In diesem Sinne wird das System erweitert. Die Messung des Quantenzustands an einem Punkt beeinflusst auch den Quantenzustand an jedem anderen Punkt.

Der Beweis dafür stammt im Wesentlichen von Srednecki in einer grundlegenden Arbeit zur Quantenthermalisierung, die ich Ihnen empfehlen möchte, einen Blick darauf zu werfen (2) . Für das obige System zeigt Srednecki, dass die Eigenzustände des Systems ein Teilchenverhalten ergeben, das mit der statistischen Maxwell-Boltzmann-Mechanik übereinstimmt, und dass sich außerdem Systeme entwickeln, die weit vom Gleichgewicht entfernt sind (wie der von Ihnen erwähnte Fall, in dem alles lokalisiert beginnt). zu einem Gleichgewichtszustand, der diesen Vorhersagen gehorcht. Darüber hinaus haben nachfolgende Arbeiten betont, dass jedes System, das diese Selbstausgleichseigenschaft hat, die als Eigenzustandsthermisierung bekannt ist, zwangsläufig auch eine Volumenskalierung der Verschränkungsentropie zeigt (siehe zum Beispiel (3) ).

Dekohärenz

Alles, was ich bisher gesagt habe, betraf den reinen Quantenzustand, aber die Leute sprechen oft über diese Art von System im Sinne von Dekohärenz. Was ist die Verbindung?

Nun, Dekohärenz tritt auf, wenn das interessierende System mit vielen anderen unzugänglichen Systemen verstrickt ist – und genau das passiert hier (4, 5) . Da jeder Teil des Systems mit jedem anderen verschränkt ist, wäre es selbst für ein System von moderater Größe praktisch unmöglich, die Kohärenz zwischen verschiedenen Teilen zu beobachten. Dies bedeutet, dass das System funktional nicht von einem System ohne Kohärenz oder nur einem klassischen statistischen Ensemble zu unterscheiden ist. Das ist das Wunder der Verschränkung – wenn man genug davon hat, werden die Dinge einfacher statt komplizierter. Deshalb können Messungen, die immer einen komplizierten Verschränkungszustand zwischen System und Apparatur erzeugen, dennoch zu Erkenntnissen führen.

Fazit

Es gibt zwei gültige Möglichkeiten, wie man den Zustand der Kiste kollidierender Teilchen nach langer Zeit beschreiben kann:

  1. Es handelt sich um einen komplexen Vielteilchen-Verschränkungszustand, in dem jeder Teil gleichermaßen mit jedem anderen Teil verschränkt ist, jedoch so, dass statistische Standardergebnisse (wie die Maxwell-Boltzmann-Verteilung) für eine Einzelteilchenmessung reproduziert werden.
  2. Aufgrund der hohen Verschränkung können die Teilchen für alle praktischen Zwecke auch als dekohärierte klassische Teilchen behandelt werden, wobei sie in diesem Fall natürlich eine wohldefinierte Position und einen wohldefinierten Impuls haben.

Keine dieser Behauptungen ist falsch, und jede könnte im richtigen Kontext nützlich sein.

Weiterführende Literatur über die Beziehung zwischen Vielteilchenzuständen, Verschränkung und Thermalisierung: physical.stackexchange.com/questions/213733/… , physical.stackexchange.com/questions/248488/… , und Links darin.

Es kommt ganz auf die Randbedingungen an. Für Randbedingungen wie eine 3D-Box mit reflektierenden Wänden der anfängliche Quantenzustand Ψ bleibt ein Quantenzustand mit der einzigartigen Wellenfunktion in Abhängigkeit von Variablen jedes Teilchens:

Ψ ( r 1 , . . . , r n , t ) .
Wenn die Randbedingungen so sind, dass ein Austausch mit der Umgebung möglich ist, kann der Dichtematrix-Ansatz geeigneter sein.

In beiden Fällen werden die Positionen von Partikeln statistisch vorhergesagt, aber im letzteren Fall gibt es möglicherweise kein Interferenzphänomen (oder es wird weniger ausgeprägt sein).

Betrachten Sie Ihren Fall für ein Doppelspaltexperiment ohne und mit Teilchenpositionsmessungen, Messungen vor dem Schirm.

Um Ihr Problem genau zu lösen, müssten Sie die Schrödinger-Gleichung lösen

ich t Ψ ( r 1 r N , t ) = H   Ψ ( r 1 r N , t )

wo Ψ ( r 1 r N , t ) ist die Wellenfunktion der N Partikel u

H = ich N p ich 2 2 m + ich < j N u ich j + v ext

wo u ich j ist ein gewisses Paarpotential und v ext ist das Boxpotential. Natürlich benötigen Sie auch eine Anfangsbedingung

Ψ ( r 1 r N , 0 ) = Ψ ~ ( r 1 r N )

Das erste, was Sie beachten müssen, ist, dass es unangemessen ist, von "der Wellenfunktion jedes Teilchens" zu sprechen, da Sie die gesamte Wellenfunktion des Teilchens berücksichtigen müssen N Partikel ( Ψ ). Wenn die Teilchen nicht unterscheidbar sind, muss diese Funktion einige Symmetrien besitzen, je nachdem, welche Art von Teilchen Sie betrachten (Bosonen oder Fermionen).

Es ist wirklich schwierig, ein solches Problem analytisch oder numerisch zu lösen. Ein guter Ausgangspunkt, um eine qualitative Vorstellung zu bekommen, wäre, die entsprechenden Gleichungen für zwei Teilchen zu lösen und zu sehen, was passiert.

Dies wurde von den Autoren dieses Artikels numerisch unter Verwendung eines Gaußschen und quadratischen Potentials mit unterscheidbaren und nicht unterscheidbaren Teilchen durchgeführt. im letzteren Fall wurden symmetrisierte (bosonische) und antisymmetrisierte (fermionische) Wellenfunktionen betrachtet:

Ψ ' ( x 1 , x 2 ) = 1 2 [ Ψ ( x 1 , x 2 ) ± Ψ ( x 2 , x 1 ) ]

Die Wellenfunktion bei t = 0 wird als Produkt zweier Gaußscher Wellenpakete angenommen:

Ψ ( x 1 , x 2 , t = 0 ) = g ( x 1 , x 1 0 , k 1 , σ )   g ( x 2 , x 2 0 , k 2 , σ )

Wo

g ( x , x 0 , k ) = e ich k x e ( x x 0 ) 2 4 σ 2

und k 2 = k 1 .

In der folgenden Grafik sehen Sie beispielsweise das Ergebnis der Kollision zweier unterscheidbarer Teilchen gleicher Masse m Wechselwirkung über ein quadratisches Potential (die Kurven sind die Wahrscheinlichkeitsdichten, die durch die Wellenfunktionen erhalten werden):

Geben Sie hier die Bildbeschreibung ein

Die Zahlen in der oberen linken Ecke geben die Zeit an (in Einheiten von 100 × Zeitschritt) und die Kanten der Rahmen entsprechen den Wänden der Box.

Man sieht, dass es nach dem Stoß tatsächlich zu einer „Spreizung“ der Wellenpakete (genauer: ihrem Quadrat zum Quadrat, also den Wahrscheinlichkeitsdichten) kommt (vgl. 1 und 120).

Zitat aus dem Artikel:

In Abb. 5 zeigen wir neun Bilder aus dem Film einer abstoßenden m-m-Kollision, bei der die mittlere kinetische Energie einem Viertel der Barrierenhöhe entspricht. Es ist zu sehen, dass die anfänglichen Pakete langsamer werden, wenn sie sich einander nähern, wobei ihre gegenseitige Abstoßung die Pakete verengt und anhebt, bis zu dem Zeitpunkt (50), wenn sie beginnen, zurückzuprallen. Die Wellenpakete behalten zu noch späteren Zeiten ihre Form bei, mit einer fortschreitenden Verbreiterung, bis sie mit den Wänden kollidieren und aufbrechen.

Im Zusammenhang mit der Festkörperphysik war eine eng verwandte Fragestellung in den letzten Jahren Gegenstand aktiver Forschung. Die meisten wechselwirkenden Systeme thermalisieren (und delokalisieren) tatsächlich über lange Zeitskalen. Bestimmte Systeme, deren Unordnung viel stärker ist als ihre Wechselwirkungen, erfahren jedoch eine „Vielkörperlokalisierung“, in der die einzelnen Teilchen auf unbestimmte Zeit „stecken bleiben“. Dies hat viele makroskopische Konsequenzen, wie fehlende elektrische Leitung (weil sich die Elektronen nicht frei bewegen können) und Verschränkungsentropie, die nur als Flächengesetz und nicht als Volumengesetz für jeden Eigenzustand (nicht nur den Grundzustand) skaliert. Es gibt viel zu viele Arbeiten zu diesem Thema, um sie aufzulisten, aber die Originalarbeit, mit der alles begann, ist Basko, Aleiner und Altschuler (2006).

Ein dichtes Gas mit relativ starken Wechselwirkungen (nach Gasstandards) könnte grob als sehr stark ungeordneter, schwach wechselwirkender Festkörper angesehen werden, daher vermute ich, dass Ihr Gedankenexperiment genau wie im Festkörperfall entweder zu einem delokalisierten oder führen könnte ein lokalisierter Zustand, abhängig von den Einzelheiten der Dichte des Gases und der Stärke seiner Wechselwirkungen.

Ich bin nicht davon überzeugt, dass ein dichtes Gas einer einzelnen Spezies mit starken Wechselwirkungen als Äquivalent zu statischer Unordnung angesehen werden kann. In einem Fall hat das System Translationsinvarianz, im anderen nicht. Ein interessanter Zwischenfall ist jedoch der, bei dem es zwei Arten von Teilchen gibt, eine viel schwerer als die andere, bei der man sich vorstellen könnte, dass die schweren Teilchen "fast" statisch sind und für die leichteren wie eine Unordnung aussehen. Dies wird beispielsweise hier ein wenig diskutiert: arxiv.org/abs/1606.05650 .
(aber +1 für die Erwähnung von MBL als andere Möglichkeit für ein interagierendes System)

Quanteneffekte treten auf, wenn die Teilchenkonzentration zufriedenstellend ist,

N v n q
wo N ist die Anzahl der Teilchen, v ist das Volumen, und n q ist die Quantenkonzentration , für die der Teilchenabstand gleich der thermischen De-Broglie-Wellenlänge ist, so dass sich die Wellenfunktionen der Teilchen kaum überlappen. Da die Quantenkonzentration von der Temperatur abhängt, bringen hohe Temperaturen die meisten Systeme in die klassische Grenze, es sei denn, sie haben eine sehr hohe Dichte, z. B. einen Weißen Zwerg oder das sehr frühe Universum.

Die Quantennatur des Teilchens manifestiert sich darin, dass Bosonen der Bose-Einstein-Statistik und Fermionen der Fermi-Dirac-Statistik gehorchen. Sowohl die Fermi-Dirac- als auch die Bose-Einstein-Statistik werden durch die klassische Maxwell-Boltzmann-Statistik bei hoher Temperatur und niedriger Konzentration, bei der die Quanteneffekte vernachlässigbar sind, gut angenähert.

Ich nehme an, Sie meinen, dass das Gas in einer Zauberkiste enthalten ist. Andernfalls werden die Wände Teil des Systems und tauschen Impuls/Energie mit den „Teilchen“ aus. Ich habe keine Antwort für Sie; Ich weiß nicht. Was ich weiß, ist, dass keine der Teilchen-Teilchen-Kollisionen anders als durch die Verwendung einer Wahrscheinlichkeitsverteilung charakterisiert werden kann. Der gesunde Menschenverstand verlangt, dass unter der Annahme, dass Sie die ungefähre Position und den Impuls eines Teilchens zum Zeitpunkt t = 0 kennen, und unter der Annahme, dass die Verteilung anderer Teilchen in der Box zufällig ist und sie sich alle im thermischen Gleichgewicht mit den Wänden befinden, die Position oder der Impuls mehr und sein wird zu Zeiten t>0 gleichmäßiger über das Volumen (Phasenraum) verteilt. Sagen Sie mir also, verwenden Sie das Teilchen als Ursprung der Koordinaten der Wellengleichung oder ist der Referenzrahmen durch die Box festgelegt? Soweit ich das beurteilen kann, Ihre Frage ist nicht nützlich und hat keinen Vorhersagewert. Mir scheint, Ihre Frage betrifft den Zustand der Wellengleichung zwischen den Beobachtungen. Was (vielleicht, weil ich ein Fan der Kopenhagener Interpretation bin) zutiefst fehlgeleitet erscheint. Sie können das Teilchen wahrscheinlich einmal (t = 0) lokalisieren, aber was dann? Wie beobachtest du es ein 2. Mal? Es ist klar, dass die zweite Beobachtung nicht an einem äquivalent lokalisierten Ort stattfinden wird. (abgesehen von einer Falle). einmal (t=0) aber was dann? Wie beobachtest du es ein 2. Mal? Es ist klar, dass die zweite Beobachtung nicht an einem äquivalent lokalisierten Ort stattfinden wird. (abgesehen von einer Falle). einmal (t=0) aber was dann? Wie beobachtest du es ein 2. Mal? Es ist klar, dass die zweite Beobachtung nicht an einem äquivalent lokalisierten Ort stattfinden wird. (abgesehen von einer Falle).

Das Verhalten der Moleküle in Ihrem gedanken-Experiment kann durch Verwendung von Dekohärenz angegangen werden. Aber ich glaube nicht, dass Sie eine endgültige Antwort erhalten können, bis jemand eine vollständige Simulation durchführt (oder bis die Antwort eines Experten einen formellen Beweis dafür liefern kann, was wirklich passiert, aber ich bin nicht in der Lage, das zu tun). Die Dekohärenzeffekte können heuristisch argumentiert werden, aber mit diesem Ansatz ist die Antwort, die ich gefunden habe, mehrdeutig.

Einerseits ist ein einzelnes Molekül nicht isoliert, es interagiert mit einer makroskopischen Umgebung (Gas, elektromagnetische Strahlung, Wände). In den meisten Fällen wirkt diese Umgebung so, dass sie den verschlungenen Zustand eines einzelnen Moleküls reduziert und es dazu bringt, scheinbar in eine bevorzugte Basis zu kollabieren. Die bevorzugte Basis ist normalerweise die Positionsbasis, aber dies ist nicht immer der Fall und hängt von den Details der Einrichtung ab.

Wenn die bevorzugte Basis die Positionsbasis ist, wird sich die Wellenfunktion einzelner Moleküle nicht zu lange ausbreiten, ihre Wechselwirkung mit dem Rest der Umgebung, in der sie sich befinden, lässt die Wahrscheinlichkeiten zu einem kleineren Paket "kollabieren" (wie erwähnt von P. Shor im Kommentar, es kann kein reiner Eigenzustand des Positionsoperators sein). Aber wenn die bevorzugte Basis anders ist, zum Beispiel die Impulsbasis, dann wird das Endergebnis ein weitgehend delokalisiertes Wellenpaket sein. Diese zweite Option scheint eher mit der Tatsache vereinbar zu sein, dass die Gasmoleküle, sobald sie das thermodynamische Gleichgewicht erreicht haben, den Maxwell Boltzmann erfüllen Statistiken.

Die Wellenfunktion kann niemals vollständig auf die Positionsbasis kollabieren, da dies bedeuten würde, dass der Impuls unbegrenzt wäre.
Wenn Sie vorschlagen, dass molekulare oder Container-Wechselwirkungen zu einem Zusammenbruch der Wellenfunktion führen würden, glaube ich nicht, dass das stimmt.
@ConfusinglyCuriousTheThird nein, das meine ich nicht, ich werde meine Antwort neu schreiben, um zu versuchen, sie klarer zu machen